Average value of a function over involving triple integral

Click For Summary
The average value of the function f(x,y,z) = x^2z + y^2z is calculated over the region defined by the paraboloid z = 1 - x^2 - y^2 and the plane z = 0, resulting in an answer of 1/12. The discussion highlights confusion regarding the conversion to polar coordinates, specifically how the integration boundaries relate to the function being evaluated. It clarifies that the integration should focus on the volume under the paraboloid rather than directly under the function f(x,y,z). The conversion to polar coordinates involves setting z = 1 - r^2, which aligns with the volume calculation. Ultimately, the understanding of the problem's requirements becomes clearer through this exploration.
TranscendArcu
Messages
277
Reaction score
0

Homework Statement


We define average value of function over a solid to be f = 1/Volume int int int f(x,y,z) dV

So find the average value of the function f(x,y,z) = x^2z+y^2z over the region enclosed by paraboloid z = 1-x^2-y^2 and the plane z = 0

The Attempt at a Solution


Actually, solving this (I don't think) is much of a problem. I get answer 1/12. But I can't intuitively understand what is going on when/if I convert to polar to solve this problem.

For example, I convert z = 1 - x2 - y2 and get z = 1 - r2

Why should integrating this region (multiplied by r) in the boundaries 0≤r≤1 and 0≤θ≤2*pi give the region under f(x,y,z) = x^2z+y^2z? How is the information f(x,y,z) = x^2z+y^2z incorporated into these boundaries? Or have I actually done this wrong?
 
Physics news on Phys.org
Oh! Wait! We're not trying to find the volume under f(x,y,z) = x^2z+y^2z, are we? Indeed, we want to find the volume of z = 1-r^2. Right?

Okay, I think this makes intuitive sense.
 
Question: A clock's minute hand has length 4 and its hour hand has length 3. What is the distance between the tips at the moment when it is increasing most rapidly?(Putnam Exam Question) Answer: Making assumption that both the hands moves at constant angular velocities, the answer is ## \sqrt{7} .## But don't you think this assumption is somewhat doubtful and wrong?

Similar threads

  • · Replies 9 ·
Replies
9
Views
2K
Replies
1
Views
1K
Replies
6
Views
2K
Replies
7
Views
2K
  • · Replies 26 ·
Replies
26
Views
2K
Replies
8
Views
2K
  • · Replies 2 ·
Replies
2
Views
2K
  • · Replies 8 ·
Replies
8
Views
2K
Replies
2
Views
2K
  • · Replies 1 ·
Replies
1
Views
2K